Euler Totient Property Proof

In summary, the formula for the totient function $$ \phi (mn) = \phi(m) \phi(n) \frac{d}{\phi(d)} $$ is obtained by splitting the divisors of the product of two numbers into the divisors of the first number, the divisors of the second number, and the divisors of the greatest common divisor of both numbers. This is because the maximal common divisor of two numbers is the product of the common primes to both numbers, each prime raised to the minimal power appearing in either of the numbers. A proof of this method can be given by considering two numbers $$m$$ and $$n$$ and their prime factorizations, and using the formula $$\prod_{j=1}
  • #1
hawaiifiver
56
1
Hello,

I am looking at the proof (Theorem 2.5 (b) Apostol) of $$ \phi (mn) = \phi(m) \phi(n) \frac{d}{\phi(d)} $$ where $$ d = (m, n) $$.

Can someone explain how they go from

$$ \prod_{p|mn} \left( 1 - \frac{1}{p} \right) = \frac{\prod_{p|m} \left( 1 - \frac{1}{p} \right) \prod_{p|n} \left( 1 - \frac{1}{p} \right) }{\prod_{p| (m, n)} \left( 1 - \frac{1}{p} \right)} $$

Thanks
 
Physics news on Phys.org
  • #2
hawaiifiver said:
Hello,

I am looking at the proof (Theorem 2.5 (b) Apostol) of $$ \phi (mn) = \phi(m) \phi(n) \frac{d}{\phi(d)} $$ where $$ d = (m, n) $$.

Can someone explain how they go from

$$ \prod_{p|mn} \left( 1 - \frac{1}{p} \right) = \frac{\prod_{p|m} \left( 1 - \frac{1}{p} \right) \prod_{p|n} \left( 1 - \frac{1}{p} \right) }{\prod_{p| (m, n)} \left( 1 - \frac{1}{p} \right)} $$

Thanks

You can split the divisors of a product of two numbers in divisors of first number, divisors of the second one and divisors of both. These last ones

are, of course, precisely the divisors of the g.c.d. of both numbers. This is is the reason of your formula.
 
  • #3
DonAntonio said:
You can split the divisors of a product of two numbers in divisors of first number, divisors of the second one and divisors of both. These last ones

are, of course, precisely the divisors of the g.c.d. of both numbers. This is is the reason of your formula.

Hi DonAntonio,

Thanks for the swift reply. Forgive my ignorance, but is there need for a proof of that "method"

Thanks
 
  • #4
hawaiifiver said:
Hi DonAntonio,

Thanks for the swift reply. Forgive my ignorance, but is there need for a proof of that "method"

Thanks


Well, of course. It is annoying though pretty elementary. Suppose $$m=\prod_{k=1}^r p_k^{a_k}\,\,,\,\,n=\prod_{i=1}^s q_i^{b_i}$$with [itex]\,p_k,q_i\,[/itex] prime numbers, [itex]\,a_k,b_i\in \mathbb N[/itex] . Suppose there is a natural number

$$\,z\leq k,i\,\,\,s.t.\,\,\,p_1=q_1,\ldots ,p_z=q_z\Longrightarrow d=g.c.d(m,n)=\prod_{j=1}^z p_z^{d_j}$$with [itex] \,d_j=min\{a_j,b_j\}[/itex] , or in words: the maximal common divisor of two

numbers is the product of the common primes to both numbera, each prime raised to the minimal power appearing in

either of both numbers

We thus can write now
$$m=d\prod_{k=z+1}^rp_k^{a_k}\,\,,\,\,q=d\prod_{i=z+1}^s q_i^{b_i}\Longrightarrow \varphi(mn)=mn\prod_{j=1}^z\left(1-\frac{1}{p_j}\right)\prod_{k=z+1}^r\left(1-\frac{1}{p_k}\right)\prod_{i=z+1}^s\left(1-\frac{1}{q_i}\right)$$and since
$$\prod_{j=1}^z\left(1-\frac{1}{p_j}\right)=\frac{\varphi(d)}{d}$$the formula you want follows at once.

DonAntonio
 
  • #5
for your question.

The proof you are looking at is using the fact that the Euler totient function, denoted as $$ \phi(n) $$, is a multiplicative function. This means that for relatively prime numbers $$ m $$ and $$ n $$, the totient function of their product is equal to the product of their individual totient functions, as long as the numbers are relatively prime.

In the equation you provided, $$ \prod_{p|mn} \left( 1 - \frac{1}{p} \right) $$ represents the product of all the terms of the form $$ \left( 1 - \frac{1}{p} \right) $$ for all prime numbers $$ p $$ that divide the product $$ mn $$. This product can be rewritten as the product of two separate products, one for the terms that are only factors of $$ m $$ and one for the terms that are only factors of $$ n $$. This gives us the equation:

$$ \prod_{p|mn} \left( 1 - \frac{1}{p} \right) = \prod_{p|m} \left( 1 - \frac{1}{p} \right) \prod_{p|n} \left( 1 - \frac{1}{p} \right) $$

Next, we need to consider the factors that are common to both $$ m $$ and $$ n $$. This is where the greatest common divisor, denoted as $$ (m, n) $$, comes into play. The product of all the terms of the form $$ \left( 1 - \frac{1}{p} \right) $$ for all prime numbers $$ p $$ that divide the greatest common divisor $$ (m, n) $$ can be represented as $$ \prod_{p|(m,n)} \left( 1 - \frac{1}{p} \right) $$.

So, in summary, the equation is showing that the product of all the terms of the form $$ \left( 1 - \frac{1}{p} \right) $$ for all prime numbers $$ p $$ that divide the product $$ mn $$ can be divided into two separate products, one for the terms that are only factors of $$ m $$ and one for the terms that are only factors of $$ n $$. Then, the common factors between $$ m $$ and $$ n $$ are divided out, giving
 

1. What is the Euler Totient Property?

The Euler Totient Property, also known as Euler's Theorem, states that if a and n are coprime positive integers, then a^φ(n) ≡ 1 mod n, where φ(n) is the Euler Totient Function which calculates the number of positive integers less than n that are also coprime to n.

2. How is the Euler Totient Property useful?

The Euler Totient Property is useful in number theory and cryptography, specifically in the RSA encryption algorithm. It allows for the efficient calculation of large exponents in modular arithmetic, which is essential in encryption and decryption processes.

3. Can you provide an example of the Euler Totient Property?

For example, let's say we have a=7 and n=12. Since 7 and 12 are coprime, we can use the Euler Totient Property to calculate 7^φ(12) ≡ 1 mod 12. The Euler Totient Function for 12 is φ(12) = 4, so we have 7^4 ≡ 1 mod 12. This is true since 2401 ≡ 1 mod 12.

4. How is the Euler Totient Property proved?

The proof of the Euler Totient Property involves using Euler's Theorem, which states that if a and n are coprime, then a^φ(n) ≡ 1 mod n. The proof then uses the concept of primitive roots and modular arithmetic to show that the Euler Totient Function accurately calculates the number of coprime integers less than n.

5. Are there any limitations to the Euler Totient Property?

One limitation of the Euler Totient Property is that it only holds true for coprime integers. If a and n are not coprime, then the property does not apply. Additionally, the Euler Totient Function can be difficult to calculate for very large numbers, making the use of the property challenging in some cases.

Similar threads

  • Linear and Abstract Algebra
Replies
3
Views
1K
Replies
5
Views
382
  • Linear and Abstract Algebra
Replies
1
Views
976
  • Linear and Abstract Algebra
Replies
1
Views
919
  • Set Theory, Logic, Probability, Statistics
Replies
0
Views
1K
  • Linear and Abstract Algebra
Replies
17
Views
4K
  • Differential Geometry
Replies
2
Views
588
Replies
13
Views
1K
  • Set Theory, Logic, Probability, Statistics
Replies
2
Views
1K
  • Linear and Abstract Algebra
Replies
1
Views
924
Back
Top